If the mass of the Earth is decreasing by sending debris in space, does its angular momentum also decrease? [duplicate]Do mankind and manmade activities/constructions have any effect on the rotation of the Earth?Is Earth's orbit altered by recoil from take-off/launch/recovery of aero/space vehicles?Why Sun is not able to attract mercury closer?How much mass must Earth lose to lose its Moon?If I do work on an object, does that increase or decrease its gravitational potential energy?Angular Momentum of the UniverseConservation of angular momentum and the behaviour of velocity in orbitsWhy are large planets away from the Sun and smaller planets near the Sun?Entropy and gravitational attractionAs the sun expands, will its Roche limit also expand?Why does angular momentum being constant prove Kepler's first law?Conservation of Angular Momentum — Earth-Moon System — Earth spin backwards?

What sort of mathematical problems are there in AI that people are working on?

5 cars in a roundabout traffic

Alphabet completion rate

Short and long term plans in a closed game in the Sicilian Defense

Cascading Repair Costs following Blown Head Gasket on a 2004 Subaru Outback

Fetch and print all properties of an object graph as string

Why is there no havdallah when going from Yom Tov into Shabbat?

MH370 blackbox - is it still possible to retrieve data from it?

Is there any evidence that the small canisters (10 liters) of 95% oxygen actually help with altitude sickness?

How to determine what is the correct level of detail when modelling?

Links to webpages in books

Is adding a new player (or players) a DM decision, or a group decision?

Is it damaging to turn off a small fridge for two days every week?

How to reply to small talk/random facts in a non-offensive way?

How dangerous are set-size assumptions?

Character discovers anti gravity emitters, flies a shipping container into space and docks with space station

Why aren't (poly-)cotton tents more popular?

Story-based adventure with functions and relationships

Is there a maximum distance from a planet that a moon can orbit?

Do French speakers not use the subjunctive informally?

Why is C++ initial allocation so much larger than C's?

What kind of wire should I use to pigtail an outlet?

Would a two-seat light aircaft with a landing speed of 20 knots and a top speed of 180 knots be technically possible?

When is the original BFGS algorithm still better than the Limited-Memory version?



If the mass of the Earth is decreasing by sending debris in space, does its angular momentum also decrease? [duplicate]


Do mankind and manmade activities/constructions have any effect on the rotation of the Earth?Is Earth's orbit altered by recoil from take-off/launch/recovery of aero/space vehicles?Why Sun is not able to attract mercury closer?How much mass must Earth lose to lose its Moon?If I do work on an object, does that increase or decrease its gravitational potential energy?Angular Momentum of the UniverseConservation of angular momentum and the behaviour of velocity in orbitsWhy are large planets away from the Sun and smaller planets near the Sun?Entropy and gravitational attractionAs the sun expands, will its Roche limit also expand?Why does angular momentum being constant prove Kepler's first law?Conservation of Angular Momentum — Earth-Moon System — Earth spin backwards?






.everyoneloves__top-leaderboard:empty,.everyoneloves__mid-leaderboard:empty,.everyoneloves__bot-mid-leaderboard:empty margin-bottom:0;








5












$begingroup$



This question already has an answer here:



  • Do mankind and manmade activities/constructions have any effect on the rotation of the Earth?

    3 answers



We are sending huge amount of debris into space from earth, and also very heavy satellites and rockets, then the mass of earth must be decreasing over time.
If the mass will decrease, then gravitational attraction between earth and sun must decrease, and its angular momentum must decrease, which would result in greater orbit of earth and earth being cooler over the time, but instead the temperature of earth is increasing, why is it so?










share|cite|improve this question











$endgroup$



marked as duplicate by John Rennie, Kyle Kanos, Carl Witthoft, Michael Seifert, GiorgioP Jun 17 at 15:07


This question has been asked before and already has an answer. If those answers do not fully address your question, please ask a new question.













  • 4




    $begingroup$
    Possible duplicates: Do mankind and manmade activities/constructions have any effect on the rotation of the Earth? , Is Earth's orbit altered by recoil from take-off/launch/recovery of aero/space vehicles? and links therein.
    $endgroup$
    – Qmechanic
    Jun 15 at 12:32







  • 2




    $begingroup$
    @Qmechanic I see this a lot from you. Why do you comment with possible duplicates but don't vote to close them as such? Just curious, I am sure you have a good reason.
    $endgroup$
    – Aaron Stevens
    Jun 15 at 13:17







  • 24




    $begingroup$
    @AaronStevens: It is for democratic reasons preferred that the community VTC with 5 votes rather than a mod with 1 vote.
    $endgroup$
    – Qmechanic
    Jun 15 at 13:21







  • 1




    $begingroup$
    If no one answers with some numbers (even if it is just a sphere that can fire bits of mass off, I don't expect "and if we model the drag via friction of air on the earth from the exhaust of a rocket not going straight up, given a net wind speed ....) let me know and I'll throw something together. I have bookmarked this as I'm always looking for "inspiration" for exam/interview/homework questions and there's something here. Thanks for that. However I may not check back for 5 years without you prompting me.
    $endgroup$
    – Alec Teal
    Jun 15 at 20:53






  • 1




    $begingroup$
    "We are sending huge amount of debris into space from earth". What evidence do you have for this assertion?
    $endgroup$
    – RonJohn
    Jun 16 at 22:26

















5












$begingroup$



This question already has an answer here:



  • Do mankind and manmade activities/constructions have any effect on the rotation of the Earth?

    3 answers



We are sending huge amount of debris into space from earth, and also very heavy satellites and rockets, then the mass of earth must be decreasing over time.
If the mass will decrease, then gravitational attraction between earth and sun must decrease, and its angular momentum must decrease, which would result in greater orbit of earth and earth being cooler over the time, but instead the temperature of earth is increasing, why is it so?










share|cite|improve this question











$endgroup$



marked as duplicate by John Rennie, Kyle Kanos, Carl Witthoft, Michael Seifert, GiorgioP Jun 17 at 15:07


This question has been asked before and already has an answer. If those answers do not fully address your question, please ask a new question.













  • 4




    $begingroup$
    Possible duplicates: Do mankind and manmade activities/constructions have any effect on the rotation of the Earth? , Is Earth's orbit altered by recoil from take-off/launch/recovery of aero/space vehicles? and links therein.
    $endgroup$
    – Qmechanic
    Jun 15 at 12:32







  • 2




    $begingroup$
    @Qmechanic I see this a lot from you. Why do you comment with possible duplicates but don't vote to close them as such? Just curious, I am sure you have a good reason.
    $endgroup$
    – Aaron Stevens
    Jun 15 at 13:17







  • 24




    $begingroup$
    @AaronStevens: It is for democratic reasons preferred that the community VTC with 5 votes rather than a mod with 1 vote.
    $endgroup$
    – Qmechanic
    Jun 15 at 13:21







  • 1




    $begingroup$
    If no one answers with some numbers (even if it is just a sphere that can fire bits of mass off, I don't expect "and if we model the drag via friction of air on the earth from the exhaust of a rocket not going straight up, given a net wind speed ....) let me know and I'll throw something together. I have bookmarked this as I'm always looking for "inspiration" for exam/interview/homework questions and there's something here. Thanks for that. However I may not check back for 5 years without you prompting me.
    $endgroup$
    – Alec Teal
    Jun 15 at 20:53






  • 1




    $begingroup$
    "We are sending huge amount of debris into space from earth". What evidence do you have for this assertion?
    $endgroup$
    – RonJohn
    Jun 16 at 22:26













5












5








5





$begingroup$



This question already has an answer here:



  • Do mankind and manmade activities/constructions have any effect on the rotation of the Earth?

    3 answers



We are sending huge amount of debris into space from earth, and also very heavy satellites and rockets, then the mass of earth must be decreasing over time.
If the mass will decrease, then gravitational attraction between earth and sun must decrease, and its angular momentum must decrease, which would result in greater orbit of earth and earth being cooler over the time, but instead the temperature of earth is increasing, why is it so?










share|cite|improve this question











$endgroup$





This question already has an answer here:



  • Do mankind and manmade activities/constructions have any effect on the rotation of the Earth?

    3 answers



We are sending huge amount of debris into space from earth, and also very heavy satellites and rockets, then the mass of earth must be decreasing over time.
If the mass will decrease, then gravitational attraction between earth and sun must decrease, and its angular momentum must decrease, which would result in greater orbit of earth and earth being cooler over the time, but instead the temperature of earth is increasing, why is it so?





This question already has an answer here:



  • Do mankind and manmade activities/constructions have any effect on the rotation of the Earth?

    3 answers







newtonian-gravity angular-momentum rotational-dynamics planets estimation






share|cite|improve this question















share|cite|improve this question













share|cite|improve this question




share|cite|improve this question








edited Jun 15 at 22:09









isanae

1245 bronze badges




1245 bronze badges










asked Jun 15 at 12:17









sawan kumawatsawan kumawat

1381 silver badge8 bronze badges




1381 silver badge8 bronze badges




marked as duplicate by John Rennie, Kyle Kanos, Carl Witthoft, Michael Seifert, GiorgioP Jun 17 at 15:07


This question has been asked before and already has an answer. If those answers do not fully address your question, please ask a new question.









marked as duplicate by John Rennie, Kyle Kanos, Carl Witthoft, Michael Seifert, GiorgioP Jun 17 at 15:07


This question has been asked before and already has an answer. If those answers do not fully address your question, please ask a new question.









  • 4




    $begingroup$
    Possible duplicates: Do mankind and manmade activities/constructions have any effect on the rotation of the Earth? , Is Earth's orbit altered by recoil from take-off/launch/recovery of aero/space vehicles? and links therein.
    $endgroup$
    – Qmechanic
    Jun 15 at 12:32







  • 2




    $begingroup$
    @Qmechanic I see this a lot from you. Why do you comment with possible duplicates but don't vote to close them as such? Just curious, I am sure you have a good reason.
    $endgroup$
    – Aaron Stevens
    Jun 15 at 13:17







  • 24




    $begingroup$
    @AaronStevens: It is for democratic reasons preferred that the community VTC with 5 votes rather than a mod with 1 vote.
    $endgroup$
    – Qmechanic
    Jun 15 at 13:21







  • 1




    $begingroup$
    If no one answers with some numbers (even if it is just a sphere that can fire bits of mass off, I don't expect "and if we model the drag via friction of air on the earth from the exhaust of a rocket not going straight up, given a net wind speed ....) let me know and I'll throw something together. I have bookmarked this as I'm always looking for "inspiration" for exam/interview/homework questions and there's something here. Thanks for that. However I may not check back for 5 years without you prompting me.
    $endgroup$
    – Alec Teal
    Jun 15 at 20:53






  • 1




    $begingroup$
    "We are sending huge amount of debris into space from earth". What evidence do you have for this assertion?
    $endgroup$
    – RonJohn
    Jun 16 at 22:26












  • 4




    $begingroup$
    Possible duplicates: Do mankind and manmade activities/constructions have any effect on the rotation of the Earth? , Is Earth's orbit altered by recoil from take-off/launch/recovery of aero/space vehicles? and links therein.
    $endgroup$
    – Qmechanic
    Jun 15 at 12:32







  • 2




    $begingroup$
    @Qmechanic I see this a lot from you. Why do you comment with possible duplicates but don't vote to close them as such? Just curious, I am sure you have a good reason.
    $endgroup$
    – Aaron Stevens
    Jun 15 at 13:17







  • 24




    $begingroup$
    @AaronStevens: It is for democratic reasons preferred that the community VTC with 5 votes rather than a mod with 1 vote.
    $endgroup$
    – Qmechanic
    Jun 15 at 13:21







  • 1




    $begingroup$
    If no one answers with some numbers (even if it is just a sphere that can fire bits of mass off, I don't expect "and if we model the drag via friction of air on the earth from the exhaust of a rocket not going straight up, given a net wind speed ....) let me know and I'll throw something together. I have bookmarked this as I'm always looking for "inspiration" for exam/interview/homework questions and there's something here. Thanks for that. However I may not check back for 5 years without you prompting me.
    $endgroup$
    – Alec Teal
    Jun 15 at 20:53






  • 1




    $begingroup$
    "We are sending huge amount of debris into space from earth". What evidence do you have for this assertion?
    $endgroup$
    – RonJohn
    Jun 16 at 22:26







4




4




$begingroup$
Possible duplicates: Do mankind and manmade activities/constructions have any effect on the rotation of the Earth? , Is Earth's orbit altered by recoil from take-off/launch/recovery of aero/space vehicles? and links therein.
$endgroup$
– Qmechanic
Jun 15 at 12:32





$begingroup$
Possible duplicates: Do mankind and manmade activities/constructions have any effect on the rotation of the Earth? , Is Earth's orbit altered by recoil from take-off/launch/recovery of aero/space vehicles? and links therein.
$endgroup$
– Qmechanic
Jun 15 at 12:32





2




2




$begingroup$
@Qmechanic I see this a lot from you. Why do you comment with possible duplicates but don't vote to close them as such? Just curious, I am sure you have a good reason.
$endgroup$
– Aaron Stevens
Jun 15 at 13:17





$begingroup$
@Qmechanic I see this a lot from you. Why do you comment with possible duplicates but don't vote to close them as such? Just curious, I am sure you have a good reason.
$endgroup$
– Aaron Stevens
Jun 15 at 13:17





24




24




$begingroup$
@AaronStevens: It is for democratic reasons preferred that the community VTC with 5 votes rather than a mod with 1 vote.
$endgroup$
– Qmechanic
Jun 15 at 13:21





$begingroup$
@AaronStevens: It is for democratic reasons preferred that the community VTC with 5 votes rather than a mod with 1 vote.
$endgroup$
– Qmechanic
Jun 15 at 13:21





1




1




$begingroup$
If no one answers with some numbers (even if it is just a sphere that can fire bits of mass off, I don't expect "and if we model the drag via friction of air on the earth from the exhaust of a rocket not going straight up, given a net wind speed ....) let me know and I'll throw something together. I have bookmarked this as I'm always looking for "inspiration" for exam/interview/homework questions and there's something here. Thanks for that. However I may not check back for 5 years without you prompting me.
$endgroup$
– Alec Teal
Jun 15 at 20:53




$begingroup$
If no one answers with some numbers (even if it is just a sphere that can fire bits of mass off, I don't expect "and if we model the drag via friction of air on the earth from the exhaust of a rocket not going straight up, given a net wind speed ....) let me know and I'll throw something together. I have bookmarked this as I'm always looking for "inspiration" for exam/interview/homework questions and there's something here. Thanks for that. However I may not check back for 5 years without you prompting me.
$endgroup$
– Alec Teal
Jun 15 at 20:53




1




1




$begingroup$
"We are sending huge amount of debris into space from earth". What evidence do you have for this assertion?
$endgroup$
– RonJohn
Jun 16 at 22:26




$begingroup$
"We are sending huge amount of debris into space from earth". What evidence do you have for this assertion?
$endgroup$
– RonJohn
Jun 16 at 22:26










7 Answers
7






active

oldest

votes


















13












$begingroup$

The mass of material sent into orbit may seem huge to a naïve observer, but it is nothing at all compared to the mass of the Earth and wouldn't result in any detectable change in its orbit or rotation. Meanwhile, the mass of the Earth is actually increasing as it picks up dust, meteorites and micro-meteorites on its passage around the sun. A certain amount of the upper atmosphere is gradually leaking into space, but that is more than offset by the space debris which arrives. The majority of orbiting satellites will eventually fall back to Earth.






share|cite|improve this answer











$endgroup$








  • 7




    $begingroup$
    Could you reference a source for your claim? E.g. en.wikipedia.org/wiki/Earth_mass#Variation claims the change is a net loss.
    $endgroup$
    – ivan_pozdeev
    Jun 16 at 8:18







  • 20




    $begingroup$
    @Michael That doesn't remove the need to cite appropriate sources, particularly when faced with standard resources that contradict your claims. If you can't attribute any particular source to this information, then go look for one. That's how academia works.
    $endgroup$
    – Emilio Pisanty
    Jun 16 at 11:39






  • 18




    $begingroup$
    @MichaelWalsby "There's nothing controversial about my stratements" they directly contradict Wikipedia, which is usually reasonably accurate. That's at least enough controversy that you should find a citation, instead of claiming that "any expert knows". If any expert knows this, it should be trivial to find a good source.
    $endgroup$
    – David Richerby
    Jun 16 at 14:29






  • 15




    $begingroup$
    @MichaelWalsby years of accumulated knowledge is oft fallible and incorrect. The issue with "... the information was accurate, but I usually can't remember where I first acquired it.." is that is how misinformation gets spread around as "hard facts".
    $endgroup$
    – Tanenthor
    Jun 17 at 7:16






  • 13




    $begingroup$
    @Michael You seem to be misunderstanding the (partly unwritten) ground rules of this venue. This is an academic forum, and given the conflicting evidence provided in this comment thread, you are expected to provide appropriate references for claims as nontrivial as yours. "Knowledge and experience" are not enough, and neither is being right or the lack of challenges by others - you need to cite sufficient sources, regardless. That is how this site works; if it bothers you, then it is still how this site works.
    $endgroup$
    – Emilio Pisanty
    Jun 17 at 10:09



















16












$begingroup$

The existing answers are already sufficient, but it is also important to emphasise that if a satellite is sent to Earth orbit, then the launch does not affect the Earth's orbit around the Sun, because the Earth and the satellite continue to interact gravitationally, and the angular momentum of the Earth+satellite system is unchanged.



For there to be any effect on Earth's orbit, the spacecraft needs to be sent off into deep space (i.e. so it is no longer gravitationally bound to Earth), and only a select few spacecraft have been sent there.



As mentioned previously, the total mass and momentum of the spacecraft that have made it to space is negligible compared to the inertia of the Earth's mass. Selecting down to the couple dozen spacecraft that have left the system makes that comparison even more lopsided.






share|cite|improve this answer









$endgroup$








  • 1




    $begingroup$
    I took it to mean the 24-ish hour spinning rotational motion BTW.
    $endgroup$
    – Alec Teal
    Jun 15 at 21:08






  • 1




    $begingroup$
    And it should also be noted that all our spacecraft leave almost all of their mass on Earth anyway - the fuel (and the parts they throw away). Until we have a better way of accelerating in space than "throw things out of the back side real fast", only a small fraction of any spacecraft will ever leave the Earth system.
    $endgroup$
    – Luaan
    Jun 17 at 7:23


















7












$begingroup$

The amount of debris we are sending is so small in comparison to the total mass of the Earth, it is probably less than the error involved in measuring the total mass....



So, any change in angular momentum will be "lost" due to the errors involved.






share|cite|improve this answer









$endgroup$








  • 3




    $begingroup$
    would be useful if you provided numbers and/or described how you'd estimate loss of momentum
    $endgroup$
    – aaaaaa
    Jun 16 at 4:06






  • 1




    $begingroup$
    @aaaaaa perhaps a comment that you should make to the accepted answer...
    $endgroup$
    – Solar Mike
    Jun 16 at 9:29


















6












$begingroup$

While rockets seem huge, they are mostly fuel. Most of the fuel is burned up in the atmosphere, and never leaves Earth. The boosters usually return to Earth, so only a relatively small payload remains in orbit, or leaves orbit outward.






share|cite|improve this answer









$endgroup$








  • 5




    $begingroup$
    While true, this is also quite irrelevant. Even if all the boosters and their fuel somehow did end up in space alongside the payload, their mass would still be completely insignificant compared to the mass of the Earth.
    $endgroup$
    – Ilmari Karonen
    Jun 15 at 21:55






  • 2




    $begingroup$
    insignificant, but not zero
    $endgroup$
    – Adrian Howard
    Jun 16 at 5:52






  • 1




    $begingroup$
    @IlmariKaronen It's another order of magnitude or two. Not to mention that it doesn't matter which parts end up on the ground/in the air and what makes it into orbit - payloads in orbit contribute to Earth's gravity just as much. You need to leave the Earth system, which means even smaller fraction of your spacecraft does.
    $endgroup$
    – Luaan
    Jun 17 at 7:26


















6












$begingroup$

Not only is the mass tiny (as explained in other answers), considerable amount of material is outside the equatorial plane (see image below). Objects orbit in a plane, and the angle this plane makes with the equator is called inclination. The larger the inclination angle, the less of a decrease in Earth's angular momentum will occur.



  • Zero degrees of inclination means the object stays in the plane of the equator ("equatorial orbit").

  • An inclination between 0° and 90° means the object moves in the same direction as the Earth ("prograde").

  • An inclination of 90° means the object moves purely north and south, around the poles ("polar orbit").

  • An inclination between 90° and 180° moves in the opposite direction of the Earth ("retrograde") and actually adds to the Earth's angular momentum.

  • An inclination of 180° would be a retrograde equatorial orbit.

space debris



Are there reasons for using specific orbits?



  • The only kind of orbit that stays over one place on the ground is a geostationary orbit. These must be done with a 0° inclination, because any other inclination will make the satellite's latitude move north and south through each orbit. At first, you might think a geostationary orbit would be perfect for taking pictures of things on the Earth. However, the high altitude of geostationary satellites means you are far away from your target, and you are never directly overhead anything other than what is on the equator.

  • If you want to take pictures of the entire Earth's surface (spy/mapping/weather satellites), the only orbit that includes the poles is a polar orbit. Inclinations close to 90° are often "good enough", even if they aren't purely polar.

  • So your spy satellite takes pictures of a site, but each time the angle of the sun is different and the shadows are all different. What you need is a sun-synchronous orbit. This orbit precesses exactly once each year, which means each time you take a picture, the sun will illuminate the target at the same angle in the sky. The inclination of these orbits tend to be between 90° and 100°, making them slightly retrograde.

  • Another use of a sun-synchronous orbit is to have a satellite permanently ride along the Earth's day-night terminator (the sunrise/sunset line). This means that the satellite will never be in shadow behind the Earth. Perfect if you need solar power!

  • Satellites that perform synthetic aperature radar actually benefit from a retrograde orbit.

Okay, but why not launch everything else into a 0° equatorial inclination?



  • It takes the least amount of energy (and thus fuel) when you launch at the same inclination as your launch site's latitude. That's not to say you can't launch at a different inclination; it will just take more fuel.

  • If you want a 0° inclination with the least amount of fuel, you will need to launch from the equator. That's why the Ariane rocket launched from French Guiana.

  • Kennedy Space Center (KSC) is about 29°N. Most satellites launched from there end up with an inclination of about 29°, unless there is reason for a different inclination.

  • Canaveral Air Force Base is next door to KSC, and is still used for some military launches. However, many military launches are spy satellites, which (as explained above) benefit from a polar orbit. Thus, a launch complex was built at Vandenburg Air Force Base in California. At a latitude of 35°N, it is more efficient to send spy satellites into polar orbit than Canaveral.

  • Sometimes a different inclination is needed to avoid launching over heavily populated areas. Israel launches the Shavit rocket in a retrograde orbit because that takes it over the Mediterranean rather than populated areas.

  • Russia launches at Baikonur Cosmodrome in Kazakhstan, latitude 46°N. It usually launches at inclinations in the 50s, to avoid launching over populated areas and so it will orbit/land over Russian territory. Because many of the components and supply missions for the International Space Station originate from Baikonur, the ISS has an inclination of 52°.

Thus, spacecraft vary in how much they change the Earth's angular momentum, and some actually increase it.






share|cite|improve this answer









$endgroup$












  • $begingroup$
    The Ariane rockets are not launched from the equator. Guiana Space Centre is at 5.2 °N.
    $endgroup$
    – Peter Mortensen
    Jun 17 at 7:42


















5












$begingroup$

The most important thing to grasp, as given in other answers, is the sheer difference of scale between the amount of stuff we have launched into space and the amount of planet we have. According to this previous answer we've launched on the order of tens of millions of kilograms. The mass of the planet is on the order of millions of millions of millions of millions of kilograms.



The other part of the question is about why removing this mass doesn't affect Earth's orbit. A, possibly surprising, fact is that even if you decreased the mass of the planet by a significant amount you would not actually change its orbit. That is because mass comes in numerators and denominators in the relevant equations, and cancels out.



You mention "then gravitational attraction between earth and sun must decrease". That is true. If you halved the mass of the Earth, you would get half the gravitational force on it. However, because force is mass times acceleration, when you halve force and you halve mass, you are left with the same acceleration. The other fact is that the momentum of the Earth decreases when this mass decreases. Again that is true: momentum is proportional to mass times velocity. Once again though, that does not mean that it's going any slower. There is just less of it going as fast as before.



Fundamentally, orbital radius is a function of tangential velocity and centripedal acceleration, and neither of those has changed. Therefore even if we had launched an impossibly huge proportion of the planet (assuming we put it in orbit or disperse it equally in all directions) that wouldn't cause the planet to orbit further from the sun.






share|cite|improve this answer








New contributor



Josiah is a new contributor to this site. Take care in asking for clarification, commenting, and answering.
Check out our Code of Conduct.





$endgroup$








  • 2




    $begingroup$
    "mass comes in numerators and denominators in the relevant equations, and cancels out." Almost. See en.wikipedia.org/wiki/… But practically speaking, Earth's mass is very small compared to the Sun, so tiny variations of Earth's mass can safely be ignored when calculating its orbit.
    $endgroup$
    – PM 2Ring
    Jun 16 at 9:27


















5












$begingroup$

All the existing answers nicely explain the issue, but I'll add some numbers to the discussion:



In 2018, 112 objects have been launched to orbit. Let's assume a typical rocket mass of 500 t and a payload of 10 t (values for the Falcon 9). Of these 112 objects, 30 were launched into high orbits where part of the rocket exhaust may have escaped Earth's sphere of influence (SOI), and four (Elon Musk's Tesla Roadster, InSight, Parker Solar Probe, and BepiColombo) were launched into interplanetary orbits where they and their upper rocket stages, escaped Earth's SOI themselves.



For high orbits, we can assume that about half of the payload's mass leaves Earth's SOI in the form of rocket exhaust. For payloads that are launched into interplanetary space, we can assume that an additional 10 tons are sent in the form of the launcher's spent upper stage.



Assuming that 2018 is a representative year and those guesses are about correct (variations from 0.1 to 10 would be possible),



  • The total launched mass is $5 cdot 10^7 kg/year$

  • The total satellite mass increases by $1 cdot 10^6 kg/year$

  • The total satellite mass in high orbits increases by $3 cdot 10^5 kg/year$

  • The total mass leaving Earth's SOI, from both interplanetary launches and rocket exhaust of high-orbit satellites, is $3 cdot 10^5 kg/year$

As discussed in the other answers, the mass of satellites that stay inside Earth's SOI has no influence on the distance between Earth and the Sun. The only relevant part is the $3 cdot 10^5 kg/year$ that are launched away from Earth.



As you correctly noted, the mass itself isn't relevant, the impulse is, so let's assume that the mass leaves Earth at 10 km/s. The fastest-ever spacecraft to leave Earth was New Horizons at 16 km/s, while a typical Mars transfer takes 6 km/s. Rocket exhaust is typically much slower, on the order of 3 km/s.



We arrive at an impulse of $ 3 cdot 10^9 Ns / year $, or an average force of $ 100 N $.



But in which direction is the force applied? For interplanetary probes the ejection direction depends on your intended destination and transfer plan, while for rocket exhaust from satellites in high orbits it's pretty much random. All in all, I guess that it will pretty much even out. After all, in 2018 two launches were to higher solar orbits and two were to lower solar orbits.



If the force is applied in the same direction in which Earth is moving, it is accelerated and thus lifted into a higher orbit. If the force is applied in the opposite direction, it is decelerated and moves into a lower orbit (closer to the Sun).



The orbital energy of Earth is calculated as $
-frac
G cdot M cdot m_Earth2a$
, where G is the constant of gravity, M is the total mass of the Earth-Sun system, and a is the semi-major axis (average distance between Earth and Sun).



The power at which Earth is accelerated or decelerated is $fracdEdt = Fv$, where F is the force of acceleration and v is the orbital velocity of Earth. We arrive at



$a(t) = - fracG cdot M cdot m_Earth2 cdot (E_0 pm F v t) = - fracG cdot M cdot m_Earth2 cdot E_0 cdot frac11 pm cfracF vE_0 cdot t $



$fracda(t)dt = - fracG cdot M cdot m_Earth2 cdot E_0 cdot fracda(t)dt (frac11 pm fracF vE_0 cdot t) = - fracG cdot M cdot m_Earth2 cdot E_0 cdot fracmp fracF vE_0(1 pm fracF vE_0 cdot t)^2 $.



$fracF vE_0 cdot t$ is negligible compared to 1 (about $t cdot 3 cdot 10^-20 / year$), so the formula is simplified to:



$fracdadt = pm fracG cdot M cdot m_Earth2 cdot E_0 cdot fracF vE_0 = pm GM cdot fracm_Earthcdot v2 cdot E_0^2 cdot F = 1.8 cdot 10^-18 fracfracmsN cdot F$.



So the result on Earth's distance from the Sun is $pm 1.8 cdot 10^-16 m/s = pm 5 cdot 10^-9 m/year$, assuming that everything is ejected in the same direction.






share|cite|improve this answer











$endgroup$








  • 1




    $begingroup$
    Finally some numbers instead of all the hand waving!
    $endgroup$
    – Peter Mortensen
    Jun 17 at 7:46



















7 Answers
7






active

oldest

votes








7 Answers
7






active

oldest

votes









active

oldest

votes






active

oldest

votes









13












$begingroup$

The mass of material sent into orbit may seem huge to a naïve observer, but it is nothing at all compared to the mass of the Earth and wouldn't result in any detectable change in its orbit or rotation. Meanwhile, the mass of the Earth is actually increasing as it picks up dust, meteorites and micro-meteorites on its passage around the sun. A certain amount of the upper atmosphere is gradually leaking into space, but that is more than offset by the space debris which arrives. The majority of orbiting satellites will eventually fall back to Earth.






share|cite|improve this answer











$endgroup$








  • 7




    $begingroup$
    Could you reference a source for your claim? E.g. en.wikipedia.org/wiki/Earth_mass#Variation claims the change is a net loss.
    $endgroup$
    – ivan_pozdeev
    Jun 16 at 8:18







  • 20




    $begingroup$
    @Michael That doesn't remove the need to cite appropriate sources, particularly when faced with standard resources that contradict your claims. If you can't attribute any particular source to this information, then go look for one. That's how academia works.
    $endgroup$
    – Emilio Pisanty
    Jun 16 at 11:39






  • 18




    $begingroup$
    @MichaelWalsby "There's nothing controversial about my stratements" they directly contradict Wikipedia, which is usually reasonably accurate. That's at least enough controversy that you should find a citation, instead of claiming that "any expert knows". If any expert knows this, it should be trivial to find a good source.
    $endgroup$
    – David Richerby
    Jun 16 at 14:29






  • 15




    $begingroup$
    @MichaelWalsby years of accumulated knowledge is oft fallible and incorrect. The issue with "... the information was accurate, but I usually can't remember where I first acquired it.." is that is how misinformation gets spread around as "hard facts".
    $endgroup$
    – Tanenthor
    Jun 17 at 7:16






  • 13




    $begingroup$
    @Michael You seem to be misunderstanding the (partly unwritten) ground rules of this venue. This is an academic forum, and given the conflicting evidence provided in this comment thread, you are expected to provide appropriate references for claims as nontrivial as yours. "Knowledge and experience" are not enough, and neither is being right or the lack of challenges by others - you need to cite sufficient sources, regardless. That is how this site works; if it bothers you, then it is still how this site works.
    $endgroup$
    – Emilio Pisanty
    Jun 17 at 10:09
















13












$begingroup$

The mass of material sent into orbit may seem huge to a naïve observer, but it is nothing at all compared to the mass of the Earth and wouldn't result in any detectable change in its orbit or rotation. Meanwhile, the mass of the Earth is actually increasing as it picks up dust, meteorites and micro-meteorites on its passage around the sun. A certain amount of the upper atmosphere is gradually leaking into space, but that is more than offset by the space debris which arrives. The majority of orbiting satellites will eventually fall back to Earth.






share|cite|improve this answer











$endgroup$








  • 7




    $begingroup$
    Could you reference a source for your claim? E.g. en.wikipedia.org/wiki/Earth_mass#Variation claims the change is a net loss.
    $endgroup$
    – ivan_pozdeev
    Jun 16 at 8:18







  • 20




    $begingroup$
    @Michael That doesn't remove the need to cite appropriate sources, particularly when faced with standard resources that contradict your claims. If you can't attribute any particular source to this information, then go look for one. That's how academia works.
    $endgroup$
    – Emilio Pisanty
    Jun 16 at 11:39






  • 18




    $begingroup$
    @MichaelWalsby "There's nothing controversial about my stratements" they directly contradict Wikipedia, which is usually reasonably accurate. That's at least enough controversy that you should find a citation, instead of claiming that "any expert knows". If any expert knows this, it should be trivial to find a good source.
    $endgroup$
    – David Richerby
    Jun 16 at 14:29






  • 15




    $begingroup$
    @MichaelWalsby years of accumulated knowledge is oft fallible and incorrect. The issue with "... the information was accurate, but I usually can't remember where I first acquired it.." is that is how misinformation gets spread around as "hard facts".
    $endgroup$
    – Tanenthor
    Jun 17 at 7:16






  • 13




    $begingroup$
    @Michael You seem to be misunderstanding the (partly unwritten) ground rules of this venue. This is an academic forum, and given the conflicting evidence provided in this comment thread, you are expected to provide appropriate references for claims as nontrivial as yours. "Knowledge and experience" are not enough, and neither is being right or the lack of challenges by others - you need to cite sufficient sources, regardless. That is how this site works; if it bothers you, then it is still how this site works.
    $endgroup$
    – Emilio Pisanty
    Jun 17 at 10:09














13












13








13





$begingroup$

The mass of material sent into orbit may seem huge to a naïve observer, but it is nothing at all compared to the mass of the Earth and wouldn't result in any detectable change in its orbit or rotation. Meanwhile, the mass of the Earth is actually increasing as it picks up dust, meteorites and micro-meteorites on its passage around the sun. A certain amount of the upper atmosphere is gradually leaking into space, but that is more than offset by the space debris which arrives. The majority of orbiting satellites will eventually fall back to Earth.






share|cite|improve this answer











$endgroup$



The mass of material sent into orbit may seem huge to a naïve observer, but it is nothing at all compared to the mass of the Earth and wouldn't result in any detectable change in its orbit or rotation. Meanwhile, the mass of the Earth is actually increasing as it picks up dust, meteorites and micro-meteorites on its passage around the sun. A certain amount of the upper atmosphere is gradually leaking into space, but that is more than offset by the space debris which arrives. The majority of orbiting satellites will eventually fall back to Earth.







share|cite|improve this answer














share|cite|improve this answer



share|cite|improve this answer








edited Jun 15 at 13:56

























answered Jun 15 at 12:45









Michael WalsbyMichael Walsby

5161 silver badge6 bronze badges




5161 silver badge6 bronze badges







  • 7




    $begingroup$
    Could you reference a source for your claim? E.g. en.wikipedia.org/wiki/Earth_mass#Variation claims the change is a net loss.
    $endgroup$
    – ivan_pozdeev
    Jun 16 at 8:18







  • 20




    $begingroup$
    @Michael That doesn't remove the need to cite appropriate sources, particularly when faced with standard resources that contradict your claims. If you can't attribute any particular source to this information, then go look for one. That's how academia works.
    $endgroup$
    – Emilio Pisanty
    Jun 16 at 11:39






  • 18




    $begingroup$
    @MichaelWalsby "There's nothing controversial about my stratements" they directly contradict Wikipedia, which is usually reasonably accurate. That's at least enough controversy that you should find a citation, instead of claiming that "any expert knows". If any expert knows this, it should be trivial to find a good source.
    $endgroup$
    – David Richerby
    Jun 16 at 14:29






  • 15




    $begingroup$
    @MichaelWalsby years of accumulated knowledge is oft fallible and incorrect. The issue with "... the information was accurate, but I usually can't remember where I first acquired it.." is that is how misinformation gets spread around as "hard facts".
    $endgroup$
    – Tanenthor
    Jun 17 at 7:16






  • 13




    $begingroup$
    @Michael You seem to be misunderstanding the (partly unwritten) ground rules of this venue. This is an academic forum, and given the conflicting evidence provided in this comment thread, you are expected to provide appropriate references for claims as nontrivial as yours. "Knowledge and experience" are not enough, and neither is being right or the lack of challenges by others - you need to cite sufficient sources, regardless. That is how this site works; if it bothers you, then it is still how this site works.
    $endgroup$
    – Emilio Pisanty
    Jun 17 at 10:09













  • 7




    $begingroup$
    Could you reference a source for your claim? E.g. en.wikipedia.org/wiki/Earth_mass#Variation claims the change is a net loss.
    $endgroup$
    – ivan_pozdeev
    Jun 16 at 8:18







  • 20




    $begingroup$
    @Michael That doesn't remove the need to cite appropriate sources, particularly when faced with standard resources that contradict your claims. If you can't attribute any particular source to this information, then go look for one. That's how academia works.
    $endgroup$
    – Emilio Pisanty
    Jun 16 at 11:39






  • 18




    $begingroup$
    @MichaelWalsby "There's nothing controversial about my stratements" they directly contradict Wikipedia, which is usually reasonably accurate. That's at least enough controversy that you should find a citation, instead of claiming that "any expert knows". If any expert knows this, it should be trivial to find a good source.
    $endgroup$
    – David Richerby
    Jun 16 at 14:29






  • 15




    $begingroup$
    @MichaelWalsby years of accumulated knowledge is oft fallible and incorrect. The issue with "... the information was accurate, but I usually can't remember where I first acquired it.." is that is how misinformation gets spread around as "hard facts".
    $endgroup$
    – Tanenthor
    Jun 17 at 7:16






  • 13




    $begingroup$
    @Michael You seem to be misunderstanding the (partly unwritten) ground rules of this venue. This is an academic forum, and given the conflicting evidence provided in this comment thread, you are expected to provide appropriate references for claims as nontrivial as yours. "Knowledge and experience" are not enough, and neither is being right or the lack of challenges by others - you need to cite sufficient sources, regardless. That is how this site works; if it bothers you, then it is still how this site works.
    $endgroup$
    – Emilio Pisanty
    Jun 17 at 10:09








7




7




$begingroup$
Could you reference a source for your claim? E.g. en.wikipedia.org/wiki/Earth_mass#Variation claims the change is a net loss.
$endgroup$
– ivan_pozdeev
Jun 16 at 8:18





$begingroup$
Could you reference a source for your claim? E.g. en.wikipedia.org/wiki/Earth_mass#Variation claims the change is a net loss.
$endgroup$
– ivan_pozdeev
Jun 16 at 8:18





20




20




$begingroup$
@Michael That doesn't remove the need to cite appropriate sources, particularly when faced with standard resources that contradict your claims. If you can't attribute any particular source to this information, then go look for one. That's how academia works.
$endgroup$
– Emilio Pisanty
Jun 16 at 11:39




$begingroup$
@Michael That doesn't remove the need to cite appropriate sources, particularly when faced with standard resources that contradict your claims. If you can't attribute any particular source to this information, then go look for one. That's how academia works.
$endgroup$
– Emilio Pisanty
Jun 16 at 11:39




18




18




$begingroup$
@MichaelWalsby "There's nothing controversial about my stratements" they directly contradict Wikipedia, which is usually reasonably accurate. That's at least enough controversy that you should find a citation, instead of claiming that "any expert knows". If any expert knows this, it should be trivial to find a good source.
$endgroup$
– David Richerby
Jun 16 at 14:29




$begingroup$
@MichaelWalsby "There's nothing controversial about my stratements" they directly contradict Wikipedia, which is usually reasonably accurate. That's at least enough controversy that you should find a citation, instead of claiming that "any expert knows". If any expert knows this, it should be trivial to find a good source.
$endgroup$
– David Richerby
Jun 16 at 14:29




15




15




$begingroup$
@MichaelWalsby years of accumulated knowledge is oft fallible and incorrect. The issue with "... the information was accurate, but I usually can't remember where I first acquired it.." is that is how misinformation gets spread around as "hard facts".
$endgroup$
– Tanenthor
Jun 17 at 7:16




$begingroup$
@MichaelWalsby years of accumulated knowledge is oft fallible and incorrect. The issue with "... the information was accurate, but I usually can't remember where I first acquired it.." is that is how misinformation gets spread around as "hard facts".
$endgroup$
– Tanenthor
Jun 17 at 7:16




13




13




$begingroup$
@Michael You seem to be misunderstanding the (partly unwritten) ground rules of this venue. This is an academic forum, and given the conflicting evidence provided in this comment thread, you are expected to provide appropriate references for claims as nontrivial as yours. "Knowledge and experience" are not enough, and neither is being right or the lack of challenges by others - you need to cite sufficient sources, regardless. That is how this site works; if it bothers you, then it is still how this site works.
$endgroup$
– Emilio Pisanty
Jun 17 at 10:09





$begingroup$
@Michael You seem to be misunderstanding the (partly unwritten) ground rules of this venue. This is an academic forum, and given the conflicting evidence provided in this comment thread, you are expected to provide appropriate references for claims as nontrivial as yours. "Knowledge and experience" are not enough, and neither is being right or the lack of challenges by others - you need to cite sufficient sources, regardless. That is how this site works; if it bothers you, then it is still how this site works.
$endgroup$
– Emilio Pisanty
Jun 17 at 10:09














16












$begingroup$

The existing answers are already sufficient, but it is also important to emphasise that if a satellite is sent to Earth orbit, then the launch does not affect the Earth's orbit around the Sun, because the Earth and the satellite continue to interact gravitationally, and the angular momentum of the Earth+satellite system is unchanged.



For there to be any effect on Earth's orbit, the spacecraft needs to be sent off into deep space (i.e. so it is no longer gravitationally bound to Earth), and only a select few spacecraft have been sent there.



As mentioned previously, the total mass and momentum of the spacecraft that have made it to space is negligible compared to the inertia of the Earth's mass. Selecting down to the couple dozen spacecraft that have left the system makes that comparison even more lopsided.






share|cite|improve this answer









$endgroup$








  • 1




    $begingroup$
    I took it to mean the 24-ish hour spinning rotational motion BTW.
    $endgroup$
    – Alec Teal
    Jun 15 at 21:08






  • 1




    $begingroup$
    And it should also be noted that all our spacecraft leave almost all of their mass on Earth anyway - the fuel (and the parts they throw away). Until we have a better way of accelerating in space than "throw things out of the back side real fast", only a small fraction of any spacecraft will ever leave the Earth system.
    $endgroup$
    – Luaan
    Jun 17 at 7:23















16












$begingroup$

The existing answers are already sufficient, but it is also important to emphasise that if a satellite is sent to Earth orbit, then the launch does not affect the Earth's orbit around the Sun, because the Earth and the satellite continue to interact gravitationally, and the angular momentum of the Earth+satellite system is unchanged.



For there to be any effect on Earth's orbit, the spacecraft needs to be sent off into deep space (i.e. so it is no longer gravitationally bound to Earth), and only a select few spacecraft have been sent there.



As mentioned previously, the total mass and momentum of the spacecraft that have made it to space is negligible compared to the inertia of the Earth's mass. Selecting down to the couple dozen spacecraft that have left the system makes that comparison even more lopsided.






share|cite|improve this answer









$endgroup$








  • 1




    $begingroup$
    I took it to mean the 24-ish hour spinning rotational motion BTW.
    $endgroup$
    – Alec Teal
    Jun 15 at 21:08






  • 1




    $begingroup$
    And it should also be noted that all our spacecraft leave almost all of their mass on Earth anyway - the fuel (and the parts they throw away). Until we have a better way of accelerating in space than "throw things out of the back side real fast", only a small fraction of any spacecraft will ever leave the Earth system.
    $endgroup$
    – Luaan
    Jun 17 at 7:23













16












16








16





$begingroup$

The existing answers are already sufficient, but it is also important to emphasise that if a satellite is sent to Earth orbit, then the launch does not affect the Earth's orbit around the Sun, because the Earth and the satellite continue to interact gravitationally, and the angular momentum of the Earth+satellite system is unchanged.



For there to be any effect on Earth's orbit, the spacecraft needs to be sent off into deep space (i.e. so it is no longer gravitationally bound to Earth), and only a select few spacecraft have been sent there.



As mentioned previously, the total mass and momentum of the spacecraft that have made it to space is negligible compared to the inertia of the Earth's mass. Selecting down to the couple dozen spacecraft that have left the system makes that comparison even more lopsided.






share|cite|improve this answer









$endgroup$



The existing answers are already sufficient, but it is also important to emphasise that if a satellite is sent to Earth orbit, then the launch does not affect the Earth's orbit around the Sun, because the Earth and the satellite continue to interact gravitationally, and the angular momentum of the Earth+satellite system is unchanged.



For there to be any effect on Earth's orbit, the spacecraft needs to be sent off into deep space (i.e. so it is no longer gravitationally bound to Earth), and only a select few spacecraft have been sent there.



As mentioned previously, the total mass and momentum of the spacecraft that have made it to space is negligible compared to the inertia of the Earth's mass. Selecting down to the couple dozen spacecraft that have left the system makes that comparison even more lopsided.







share|cite|improve this answer












share|cite|improve this answer



share|cite|improve this answer










answered Jun 15 at 15:04









Emilio PisantyEmilio Pisanty

88.7k23 gold badges225 silver badges458 bronze badges




88.7k23 gold badges225 silver badges458 bronze badges







  • 1




    $begingroup$
    I took it to mean the 24-ish hour spinning rotational motion BTW.
    $endgroup$
    – Alec Teal
    Jun 15 at 21:08






  • 1




    $begingroup$
    And it should also be noted that all our spacecraft leave almost all of their mass on Earth anyway - the fuel (and the parts they throw away). Until we have a better way of accelerating in space than "throw things out of the back side real fast", only a small fraction of any spacecraft will ever leave the Earth system.
    $endgroup$
    – Luaan
    Jun 17 at 7:23












  • 1




    $begingroup$
    I took it to mean the 24-ish hour spinning rotational motion BTW.
    $endgroup$
    – Alec Teal
    Jun 15 at 21:08






  • 1




    $begingroup$
    And it should also be noted that all our spacecraft leave almost all of their mass on Earth anyway - the fuel (and the parts they throw away). Until we have a better way of accelerating in space than "throw things out of the back side real fast", only a small fraction of any spacecraft will ever leave the Earth system.
    $endgroup$
    – Luaan
    Jun 17 at 7:23







1




1




$begingroup$
I took it to mean the 24-ish hour spinning rotational motion BTW.
$endgroup$
– Alec Teal
Jun 15 at 21:08




$begingroup$
I took it to mean the 24-ish hour spinning rotational motion BTW.
$endgroup$
– Alec Teal
Jun 15 at 21:08




1




1




$begingroup$
And it should also be noted that all our spacecraft leave almost all of their mass on Earth anyway - the fuel (and the parts they throw away). Until we have a better way of accelerating in space than "throw things out of the back side real fast", only a small fraction of any spacecraft will ever leave the Earth system.
$endgroup$
– Luaan
Jun 17 at 7:23




$begingroup$
And it should also be noted that all our spacecraft leave almost all of their mass on Earth anyway - the fuel (and the parts they throw away). Until we have a better way of accelerating in space than "throw things out of the back side real fast", only a small fraction of any spacecraft will ever leave the Earth system.
$endgroup$
– Luaan
Jun 17 at 7:23











7












$begingroup$

The amount of debris we are sending is so small in comparison to the total mass of the Earth, it is probably less than the error involved in measuring the total mass....



So, any change in angular momentum will be "lost" due to the errors involved.






share|cite|improve this answer









$endgroup$








  • 3




    $begingroup$
    would be useful if you provided numbers and/or described how you'd estimate loss of momentum
    $endgroup$
    – aaaaaa
    Jun 16 at 4:06






  • 1




    $begingroup$
    @aaaaaa perhaps a comment that you should make to the accepted answer...
    $endgroup$
    – Solar Mike
    Jun 16 at 9:29















7












$begingroup$

The amount of debris we are sending is so small in comparison to the total mass of the Earth, it is probably less than the error involved in measuring the total mass....



So, any change in angular momentum will be "lost" due to the errors involved.






share|cite|improve this answer









$endgroup$








  • 3




    $begingroup$
    would be useful if you provided numbers and/or described how you'd estimate loss of momentum
    $endgroup$
    – aaaaaa
    Jun 16 at 4:06






  • 1




    $begingroup$
    @aaaaaa perhaps a comment that you should make to the accepted answer...
    $endgroup$
    – Solar Mike
    Jun 16 at 9:29













7












7








7





$begingroup$

The amount of debris we are sending is so small in comparison to the total mass of the Earth, it is probably less than the error involved in measuring the total mass....



So, any change in angular momentum will be "lost" due to the errors involved.






share|cite|improve this answer









$endgroup$



The amount of debris we are sending is so small in comparison to the total mass of the Earth, it is probably less than the error involved in measuring the total mass....



So, any change in angular momentum will be "lost" due to the errors involved.







share|cite|improve this answer












share|cite|improve this answer



share|cite|improve this answer










answered Jun 15 at 12:35









Solar MikeSolar Mike

4431 silver badge7 bronze badges




4431 silver badge7 bronze badges







  • 3




    $begingroup$
    would be useful if you provided numbers and/or described how you'd estimate loss of momentum
    $endgroup$
    – aaaaaa
    Jun 16 at 4:06






  • 1




    $begingroup$
    @aaaaaa perhaps a comment that you should make to the accepted answer...
    $endgroup$
    – Solar Mike
    Jun 16 at 9:29












  • 3




    $begingroup$
    would be useful if you provided numbers and/or described how you'd estimate loss of momentum
    $endgroup$
    – aaaaaa
    Jun 16 at 4:06






  • 1




    $begingroup$
    @aaaaaa perhaps a comment that you should make to the accepted answer...
    $endgroup$
    – Solar Mike
    Jun 16 at 9:29







3




3




$begingroup$
would be useful if you provided numbers and/or described how you'd estimate loss of momentum
$endgroup$
– aaaaaa
Jun 16 at 4:06




$begingroup$
would be useful if you provided numbers and/or described how you'd estimate loss of momentum
$endgroup$
– aaaaaa
Jun 16 at 4:06




1




1




$begingroup$
@aaaaaa perhaps a comment that you should make to the accepted answer...
$endgroup$
– Solar Mike
Jun 16 at 9:29




$begingroup$
@aaaaaa perhaps a comment that you should make to the accepted answer...
$endgroup$
– Solar Mike
Jun 16 at 9:29











6












$begingroup$

While rockets seem huge, they are mostly fuel. Most of the fuel is burned up in the atmosphere, and never leaves Earth. The boosters usually return to Earth, so only a relatively small payload remains in orbit, or leaves orbit outward.






share|cite|improve this answer









$endgroup$








  • 5




    $begingroup$
    While true, this is also quite irrelevant. Even if all the boosters and their fuel somehow did end up in space alongside the payload, their mass would still be completely insignificant compared to the mass of the Earth.
    $endgroup$
    – Ilmari Karonen
    Jun 15 at 21:55






  • 2




    $begingroup$
    insignificant, but not zero
    $endgroup$
    – Adrian Howard
    Jun 16 at 5:52






  • 1




    $begingroup$
    @IlmariKaronen It's another order of magnitude or two. Not to mention that it doesn't matter which parts end up on the ground/in the air and what makes it into orbit - payloads in orbit contribute to Earth's gravity just as much. You need to leave the Earth system, which means even smaller fraction of your spacecraft does.
    $endgroup$
    – Luaan
    Jun 17 at 7:26















6












$begingroup$

While rockets seem huge, they are mostly fuel. Most of the fuel is burned up in the atmosphere, and never leaves Earth. The boosters usually return to Earth, so only a relatively small payload remains in orbit, or leaves orbit outward.






share|cite|improve this answer









$endgroup$








  • 5




    $begingroup$
    While true, this is also quite irrelevant. Even if all the boosters and their fuel somehow did end up in space alongside the payload, their mass would still be completely insignificant compared to the mass of the Earth.
    $endgroup$
    – Ilmari Karonen
    Jun 15 at 21:55






  • 2




    $begingroup$
    insignificant, but not zero
    $endgroup$
    – Adrian Howard
    Jun 16 at 5:52






  • 1




    $begingroup$
    @IlmariKaronen It's another order of magnitude or two. Not to mention that it doesn't matter which parts end up on the ground/in the air and what makes it into orbit - payloads in orbit contribute to Earth's gravity just as much. You need to leave the Earth system, which means even smaller fraction of your spacecraft does.
    $endgroup$
    – Luaan
    Jun 17 at 7:26













6












6








6





$begingroup$

While rockets seem huge, they are mostly fuel. Most of the fuel is burned up in the atmosphere, and never leaves Earth. The boosters usually return to Earth, so only a relatively small payload remains in orbit, or leaves orbit outward.






share|cite|improve this answer









$endgroup$



While rockets seem huge, they are mostly fuel. Most of the fuel is burned up in the atmosphere, and never leaves Earth. The boosters usually return to Earth, so only a relatively small payload remains in orbit, or leaves orbit outward.







share|cite|improve this answer












share|cite|improve this answer



share|cite|improve this answer










answered Jun 15 at 14:04









Adrian HowardAdrian Howard

2966 bronze badges




2966 bronze badges







  • 5




    $begingroup$
    While true, this is also quite irrelevant. Even if all the boosters and their fuel somehow did end up in space alongside the payload, their mass would still be completely insignificant compared to the mass of the Earth.
    $endgroup$
    – Ilmari Karonen
    Jun 15 at 21:55






  • 2




    $begingroup$
    insignificant, but not zero
    $endgroup$
    – Adrian Howard
    Jun 16 at 5:52






  • 1




    $begingroup$
    @IlmariKaronen It's another order of magnitude or two. Not to mention that it doesn't matter which parts end up on the ground/in the air and what makes it into orbit - payloads in orbit contribute to Earth's gravity just as much. You need to leave the Earth system, which means even smaller fraction of your spacecraft does.
    $endgroup$
    – Luaan
    Jun 17 at 7:26












  • 5




    $begingroup$
    While true, this is also quite irrelevant. Even if all the boosters and their fuel somehow did end up in space alongside the payload, their mass would still be completely insignificant compared to the mass of the Earth.
    $endgroup$
    – Ilmari Karonen
    Jun 15 at 21:55






  • 2




    $begingroup$
    insignificant, but not zero
    $endgroup$
    – Adrian Howard
    Jun 16 at 5:52






  • 1




    $begingroup$
    @IlmariKaronen It's another order of magnitude or two. Not to mention that it doesn't matter which parts end up on the ground/in the air and what makes it into orbit - payloads in orbit contribute to Earth's gravity just as much. You need to leave the Earth system, which means even smaller fraction of your spacecraft does.
    $endgroup$
    – Luaan
    Jun 17 at 7:26







5




5




$begingroup$
While true, this is also quite irrelevant. Even if all the boosters and their fuel somehow did end up in space alongside the payload, their mass would still be completely insignificant compared to the mass of the Earth.
$endgroup$
– Ilmari Karonen
Jun 15 at 21:55




$begingroup$
While true, this is also quite irrelevant. Even if all the boosters and their fuel somehow did end up in space alongside the payload, their mass would still be completely insignificant compared to the mass of the Earth.
$endgroup$
– Ilmari Karonen
Jun 15 at 21:55




2




2




$begingroup$
insignificant, but not zero
$endgroup$
– Adrian Howard
Jun 16 at 5:52




$begingroup$
insignificant, but not zero
$endgroup$
– Adrian Howard
Jun 16 at 5:52




1




1




$begingroup$
@IlmariKaronen It's another order of magnitude or two. Not to mention that it doesn't matter which parts end up on the ground/in the air and what makes it into orbit - payloads in orbit contribute to Earth's gravity just as much. You need to leave the Earth system, which means even smaller fraction of your spacecraft does.
$endgroup$
– Luaan
Jun 17 at 7:26




$begingroup$
@IlmariKaronen It's another order of magnitude or two. Not to mention that it doesn't matter which parts end up on the ground/in the air and what makes it into orbit - payloads in orbit contribute to Earth's gravity just as much. You need to leave the Earth system, which means even smaller fraction of your spacecraft does.
$endgroup$
– Luaan
Jun 17 at 7:26











6












$begingroup$

Not only is the mass tiny (as explained in other answers), considerable amount of material is outside the equatorial plane (see image below). Objects orbit in a plane, and the angle this plane makes with the equator is called inclination. The larger the inclination angle, the less of a decrease in Earth's angular momentum will occur.



  • Zero degrees of inclination means the object stays in the plane of the equator ("equatorial orbit").

  • An inclination between 0° and 90° means the object moves in the same direction as the Earth ("prograde").

  • An inclination of 90° means the object moves purely north and south, around the poles ("polar orbit").

  • An inclination between 90° and 180° moves in the opposite direction of the Earth ("retrograde") and actually adds to the Earth's angular momentum.

  • An inclination of 180° would be a retrograde equatorial orbit.

space debris



Are there reasons for using specific orbits?



  • The only kind of orbit that stays over one place on the ground is a geostationary orbit. These must be done with a 0° inclination, because any other inclination will make the satellite's latitude move north and south through each orbit. At first, you might think a geostationary orbit would be perfect for taking pictures of things on the Earth. However, the high altitude of geostationary satellites means you are far away from your target, and you are never directly overhead anything other than what is on the equator.

  • If you want to take pictures of the entire Earth's surface (spy/mapping/weather satellites), the only orbit that includes the poles is a polar orbit. Inclinations close to 90° are often "good enough", even if they aren't purely polar.

  • So your spy satellite takes pictures of a site, but each time the angle of the sun is different and the shadows are all different. What you need is a sun-synchronous orbit. This orbit precesses exactly once each year, which means each time you take a picture, the sun will illuminate the target at the same angle in the sky. The inclination of these orbits tend to be between 90° and 100°, making them slightly retrograde.

  • Another use of a sun-synchronous orbit is to have a satellite permanently ride along the Earth's day-night terminator (the sunrise/sunset line). This means that the satellite will never be in shadow behind the Earth. Perfect if you need solar power!

  • Satellites that perform synthetic aperature radar actually benefit from a retrograde orbit.

Okay, but why not launch everything else into a 0° equatorial inclination?



  • It takes the least amount of energy (and thus fuel) when you launch at the same inclination as your launch site's latitude. That's not to say you can't launch at a different inclination; it will just take more fuel.

  • If you want a 0° inclination with the least amount of fuel, you will need to launch from the equator. That's why the Ariane rocket launched from French Guiana.

  • Kennedy Space Center (KSC) is about 29°N. Most satellites launched from there end up with an inclination of about 29°, unless there is reason for a different inclination.

  • Canaveral Air Force Base is next door to KSC, and is still used for some military launches. However, many military launches are spy satellites, which (as explained above) benefit from a polar orbit. Thus, a launch complex was built at Vandenburg Air Force Base in California. At a latitude of 35°N, it is more efficient to send spy satellites into polar orbit than Canaveral.

  • Sometimes a different inclination is needed to avoid launching over heavily populated areas. Israel launches the Shavit rocket in a retrograde orbit because that takes it over the Mediterranean rather than populated areas.

  • Russia launches at Baikonur Cosmodrome in Kazakhstan, latitude 46°N. It usually launches at inclinations in the 50s, to avoid launching over populated areas and so it will orbit/land over Russian territory. Because many of the components and supply missions for the International Space Station originate from Baikonur, the ISS has an inclination of 52°.

Thus, spacecraft vary in how much they change the Earth's angular momentum, and some actually increase it.






share|cite|improve this answer









$endgroup$












  • $begingroup$
    The Ariane rockets are not launched from the equator. Guiana Space Centre is at 5.2 °N.
    $endgroup$
    – Peter Mortensen
    Jun 17 at 7:42















6












$begingroup$

Not only is the mass tiny (as explained in other answers), considerable amount of material is outside the equatorial plane (see image below). Objects orbit in a plane, and the angle this plane makes with the equator is called inclination. The larger the inclination angle, the less of a decrease in Earth's angular momentum will occur.



  • Zero degrees of inclination means the object stays in the plane of the equator ("equatorial orbit").

  • An inclination between 0° and 90° means the object moves in the same direction as the Earth ("prograde").

  • An inclination of 90° means the object moves purely north and south, around the poles ("polar orbit").

  • An inclination between 90° and 180° moves in the opposite direction of the Earth ("retrograde") and actually adds to the Earth's angular momentum.

  • An inclination of 180° would be a retrograde equatorial orbit.

space debris



Are there reasons for using specific orbits?



  • The only kind of orbit that stays over one place on the ground is a geostationary orbit. These must be done with a 0° inclination, because any other inclination will make the satellite's latitude move north and south through each orbit. At first, you might think a geostationary orbit would be perfect for taking pictures of things on the Earth. However, the high altitude of geostationary satellites means you are far away from your target, and you are never directly overhead anything other than what is on the equator.

  • If you want to take pictures of the entire Earth's surface (spy/mapping/weather satellites), the only orbit that includes the poles is a polar orbit. Inclinations close to 90° are often "good enough", even if they aren't purely polar.

  • So your spy satellite takes pictures of a site, but each time the angle of the sun is different and the shadows are all different. What you need is a sun-synchronous orbit. This orbit precesses exactly once each year, which means each time you take a picture, the sun will illuminate the target at the same angle in the sky. The inclination of these orbits tend to be between 90° and 100°, making them slightly retrograde.

  • Another use of a sun-synchronous orbit is to have a satellite permanently ride along the Earth's day-night terminator (the sunrise/sunset line). This means that the satellite will never be in shadow behind the Earth. Perfect if you need solar power!

  • Satellites that perform synthetic aperature radar actually benefit from a retrograde orbit.

Okay, but why not launch everything else into a 0° equatorial inclination?



  • It takes the least amount of energy (and thus fuel) when you launch at the same inclination as your launch site's latitude. That's not to say you can't launch at a different inclination; it will just take more fuel.

  • If you want a 0° inclination with the least amount of fuel, you will need to launch from the equator. That's why the Ariane rocket launched from French Guiana.

  • Kennedy Space Center (KSC) is about 29°N. Most satellites launched from there end up with an inclination of about 29°, unless there is reason for a different inclination.

  • Canaveral Air Force Base is next door to KSC, and is still used for some military launches. However, many military launches are spy satellites, which (as explained above) benefit from a polar orbit. Thus, a launch complex was built at Vandenburg Air Force Base in California. At a latitude of 35°N, it is more efficient to send spy satellites into polar orbit than Canaveral.

  • Sometimes a different inclination is needed to avoid launching over heavily populated areas. Israel launches the Shavit rocket in a retrograde orbit because that takes it over the Mediterranean rather than populated areas.

  • Russia launches at Baikonur Cosmodrome in Kazakhstan, latitude 46°N. It usually launches at inclinations in the 50s, to avoid launching over populated areas and so it will orbit/land over Russian territory. Because many of the components and supply missions for the International Space Station originate from Baikonur, the ISS has an inclination of 52°.

Thus, spacecraft vary in how much they change the Earth's angular momentum, and some actually increase it.






share|cite|improve this answer









$endgroup$












  • $begingroup$
    The Ariane rockets are not launched from the equator. Guiana Space Centre is at 5.2 °N.
    $endgroup$
    – Peter Mortensen
    Jun 17 at 7:42













6












6








6





$begingroup$

Not only is the mass tiny (as explained in other answers), considerable amount of material is outside the equatorial plane (see image below). Objects orbit in a plane, and the angle this plane makes with the equator is called inclination. The larger the inclination angle, the less of a decrease in Earth's angular momentum will occur.



  • Zero degrees of inclination means the object stays in the plane of the equator ("equatorial orbit").

  • An inclination between 0° and 90° means the object moves in the same direction as the Earth ("prograde").

  • An inclination of 90° means the object moves purely north and south, around the poles ("polar orbit").

  • An inclination between 90° and 180° moves in the opposite direction of the Earth ("retrograde") and actually adds to the Earth's angular momentum.

  • An inclination of 180° would be a retrograde equatorial orbit.

space debris



Are there reasons for using specific orbits?



  • The only kind of orbit that stays over one place on the ground is a geostationary orbit. These must be done with a 0° inclination, because any other inclination will make the satellite's latitude move north and south through each orbit. At first, you might think a geostationary orbit would be perfect for taking pictures of things on the Earth. However, the high altitude of geostationary satellites means you are far away from your target, and you are never directly overhead anything other than what is on the equator.

  • If you want to take pictures of the entire Earth's surface (spy/mapping/weather satellites), the only orbit that includes the poles is a polar orbit. Inclinations close to 90° are often "good enough", even if they aren't purely polar.

  • So your spy satellite takes pictures of a site, but each time the angle of the sun is different and the shadows are all different. What you need is a sun-synchronous orbit. This orbit precesses exactly once each year, which means each time you take a picture, the sun will illuminate the target at the same angle in the sky. The inclination of these orbits tend to be between 90° and 100°, making them slightly retrograde.

  • Another use of a sun-synchronous orbit is to have a satellite permanently ride along the Earth's day-night terminator (the sunrise/sunset line). This means that the satellite will never be in shadow behind the Earth. Perfect if you need solar power!

  • Satellites that perform synthetic aperature radar actually benefit from a retrograde orbit.

Okay, but why not launch everything else into a 0° equatorial inclination?



  • It takes the least amount of energy (and thus fuel) when you launch at the same inclination as your launch site's latitude. That's not to say you can't launch at a different inclination; it will just take more fuel.

  • If you want a 0° inclination with the least amount of fuel, you will need to launch from the equator. That's why the Ariane rocket launched from French Guiana.

  • Kennedy Space Center (KSC) is about 29°N. Most satellites launched from there end up with an inclination of about 29°, unless there is reason for a different inclination.

  • Canaveral Air Force Base is next door to KSC, and is still used for some military launches. However, many military launches are spy satellites, which (as explained above) benefit from a polar orbit. Thus, a launch complex was built at Vandenburg Air Force Base in California. At a latitude of 35°N, it is more efficient to send spy satellites into polar orbit than Canaveral.

  • Sometimes a different inclination is needed to avoid launching over heavily populated areas. Israel launches the Shavit rocket in a retrograde orbit because that takes it over the Mediterranean rather than populated areas.

  • Russia launches at Baikonur Cosmodrome in Kazakhstan, latitude 46°N. It usually launches at inclinations in the 50s, to avoid launching over populated areas and so it will orbit/land over Russian territory. Because many of the components and supply missions for the International Space Station originate from Baikonur, the ISS has an inclination of 52°.

Thus, spacecraft vary in how much they change the Earth's angular momentum, and some actually increase it.






share|cite|improve this answer









$endgroup$



Not only is the mass tiny (as explained in other answers), considerable amount of material is outside the equatorial plane (see image below). Objects orbit in a plane, and the angle this plane makes with the equator is called inclination. The larger the inclination angle, the less of a decrease in Earth's angular momentum will occur.



  • Zero degrees of inclination means the object stays in the plane of the equator ("equatorial orbit").

  • An inclination between 0° and 90° means the object moves in the same direction as the Earth ("prograde").

  • An inclination of 90° means the object moves purely north and south, around the poles ("polar orbit").

  • An inclination between 90° and 180° moves in the opposite direction of the Earth ("retrograde") and actually adds to the Earth's angular momentum.

  • An inclination of 180° would be a retrograde equatorial orbit.

space debris



Are there reasons for using specific orbits?



  • The only kind of orbit that stays over one place on the ground is a geostationary orbit. These must be done with a 0° inclination, because any other inclination will make the satellite's latitude move north and south through each orbit. At first, you might think a geostationary orbit would be perfect for taking pictures of things on the Earth. However, the high altitude of geostationary satellites means you are far away from your target, and you are never directly overhead anything other than what is on the equator.

  • If you want to take pictures of the entire Earth's surface (spy/mapping/weather satellites), the only orbit that includes the poles is a polar orbit. Inclinations close to 90° are often "good enough", even if they aren't purely polar.

  • So your spy satellite takes pictures of a site, but each time the angle of the sun is different and the shadows are all different. What you need is a sun-synchronous orbit. This orbit precesses exactly once each year, which means each time you take a picture, the sun will illuminate the target at the same angle in the sky. The inclination of these orbits tend to be between 90° and 100°, making them slightly retrograde.

  • Another use of a sun-synchronous orbit is to have a satellite permanently ride along the Earth's day-night terminator (the sunrise/sunset line). This means that the satellite will never be in shadow behind the Earth. Perfect if you need solar power!

  • Satellites that perform synthetic aperature radar actually benefit from a retrograde orbit.

Okay, but why not launch everything else into a 0° equatorial inclination?



  • It takes the least amount of energy (and thus fuel) when you launch at the same inclination as your launch site's latitude. That's not to say you can't launch at a different inclination; it will just take more fuel.

  • If you want a 0° inclination with the least amount of fuel, you will need to launch from the equator. That's why the Ariane rocket launched from French Guiana.

  • Kennedy Space Center (KSC) is about 29°N. Most satellites launched from there end up with an inclination of about 29°, unless there is reason for a different inclination.

  • Canaveral Air Force Base is next door to KSC, and is still used for some military launches. However, many military launches are spy satellites, which (as explained above) benefit from a polar orbit. Thus, a launch complex was built at Vandenburg Air Force Base in California. At a latitude of 35°N, it is more efficient to send spy satellites into polar orbit than Canaveral.

  • Sometimes a different inclination is needed to avoid launching over heavily populated areas. Israel launches the Shavit rocket in a retrograde orbit because that takes it over the Mediterranean rather than populated areas.

  • Russia launches at Baikonur Cosmodrome in Kazakhstan, latitude 46°N. It usually launches at inclinations in the 50s, to avoid launching over populated areas and so it will orbit/land over Russian territory. Because many of the components and supply missions for the International Space Station originate from Baikonur, the ISS has an inclination of 52°.

Thus, spacecraft vary in how much they change the Earth's angular momentum, and some actually increase it.







share|cite|improve this answer












share|cite|improve this answer



share|cite|improve this answer










answered Jun 16 at 8:24









DrSheldonDrSheldon

4771 gold badge2 silver badges11 bronze badges




4771 gold badge2 silver badges11 bronze badges











  • $begingroup$
    The Ariane rockets are not launched from the equator. Guiana Space Centre is at 5.2 °N.
    $endgroup$
    – Peter Mortensen
    Jun 17 at 7:42
















  • $begingroup$
    The Ariane rockets are not launched from the equator. Guiana Space Centre is at 5.2 °N.
    $endgroup$
    – Peter Mortensen
    Jun 17 at 7:42















$begingroup$
The Ariane rockets are not launched from the equator. Guiana Space Centre is at 5.2 °N.
$endgroup$
– Peter Mortensen
Jun 17 at 7:42




$begingroup$
The Ariane rockets are not launched from the equator. Guiana Space Centre is at 5.2 °N.
$endgroup$
– Peter Mortensen
Jun 17 at 7:42











5












$begingroup$

The most important thing to grasp, as given in other answers, is the sheer difference of scale between the amount of stuff we have launched into space and the amount of planet we have. According to this previous answer we've launched on the order of tens of millions of kilograms. The mass of the planet is on the order of millions of millions of millions of millions of kilograms.



The other part of the question is about why removing this mass doesn't affect Earth's orbit. A, possibly surprising, fact is that even if you decreased the mass of the planet by a significant amount you would not actually change its orbit. That is because mass comes in numerators and denominators in the relevant equations, and cancels out.



You mention "then gravitational attraction between earth and sun must decrease". That is true. If you halved the mass of the Earth, you would get half the gravitational force on it. However, because force is mass times acceleration, when you halve force and you halve mass, you are left with the same acceleration. The other fact is that the momentum of the Earth decreases when this mass decreases. Again that is true: momentum is proportional to mass times velocity. Once again though, that does not mean that it's going any slower. There is just less of it going as fast as before.



Fundamentally, orbital radius is a function of tangential velocity and centripedal acceleration, and neither of those has changed. Therefore even if we had launched an impossibly huge proportion of the planet (assuming we put it in orbit or disperse it equally in all directions) that wouldn't cause the planet to orbit further from the sun.






share|cite|improve this answer








New contributor



Josiah is a new contributor to this site. Take care in asking for clarification, commenting, and answering.
Check out our Code of Conduct.





$endgroup$








  • 2




    $begingroup$
    "mass comes in numerators and denominators in the relevant equations, and cancels out." Almost. See en.wikipedia.org/wiki/… But practically speaking, Earth's mass is very small compared to the Sun, so tiny variations of Earth's mass can safely be ignored when calculating its orbit.
    $endgroup$
    – PM 2Ring
    Jun 16 at 9:27















5












$begingroup$

The most important thing to grasp, as given in other answers, is the sheer difference of scale between the amount of stuff we have launched into space and the amount of planet we have. According to this previous answer we've launched on the order of tens of millions of kilograms. The mass of the planet is on the order of millions of millions of millions of millions of kilograms.



The other part of the question is about why removing this mass doesn't affect Earth's orbit. A, possibly surprising, fact is that even if you decreased the mass of the planet by a significant amount you would not actually change its orbit. That is because mass comes in numerators and denominators in the relevant equations, and cancels out.



You mention "then gravitational attraction between earth and sun must decrease". That is true. If you halved the mass of the Earth, you would get half the gravitational force on it. However, because force is mass times acceleration, when you halve force and you halve mass, you are left with the same acceleration. The other fact is that the momentum of the Earth decreases when this mass decreases. Again that is true: momentum is proportional to mass times velocity. Once again though, that does not mean that it's going any slower. There is just less of it going as fast as before.



Fundamentally, orbital radius is a function of tangential velocity and centripedal acceleration, and neither of those has changed. Therefore even if we had launched an impossibly huge proportion of the planet (assuming we put it in orbit or disperse it equally in all directions) that wouldn't cause the planet to orbit further from the sun.






share|cite|improve this answer








New contributor



Josiah is a new contributor to this site. Take care in asking for clarification, commenting, and answering.
Check out our Code of Conduct.





$endgroup$








  • 2




    $begingroup$
    "mass comes in numerators and denominators in the relevant equations, and cancels out." Almost. See en.wikipedia.org/wiki/… But practically speaking, Earth's mass is very small compared to the Sun, so tiny variations of Earth's mass can safely be ignored when calculating its orbit.
    $endgroup$
    – PM 2Ring
    Jun 16 at 9:27













5












5








5





$begingroup$

The most important thing to grasp, as given in other answers, is the sheer difference of scale between the amount of stuff we have launched into space and the amount of planet we have. According to this previous answer we've launched on the order of tens of millions of kilograms. The mass of the planet is on the order of millions of millions of millions of millions of kilograms.



The other part of the question is about why removing this mass doesn't affect Earth's orbit. A, possibly surprising, fact is that even if you decreased the mass of the planet by a significant amount you would not actually change its orbit. That is because mass comes in numerators and denominators in the relevant equations, and cancels out.



You mention "then gravitational attraction between earth and sun must decrease". That is true. If you halved the mass of the Earth, you would get half the gravitational force on it. However, because force is mass times acceleration, when you halve force and you halve mass, you are left with the same acceleration. The other fact is that the momentum of the Earth decreases when this mass decreases. Again that is true: momentum is proportional to mass times velocity. Once again though, that does not mean that it's going any slower. There is just less of it going as fast as before.



Fundamentally, orbital radius is a function of tangential velocity and centripedal acceleration, and neither of those has changed. Therefore even if we had launched an impossibly huge proportion of the planet (assuming we put it in orbit or disperse it equally in all directions) that wouldn't cause the planet to orbit further from the sun.






share|cite|improve this answer








New contributor



Josiah is a new contributor to this site. Take care in asking for clarification, commenting, and answering.
Check out our Code of Conduct.





$endgroup$



The most important thing to grasp, as given in other answers, is the sheer difference of scale between the amount of stuff we have launched into space and the amount of planet we have. According to this previous answer we've launched on the order of tens of millions of kilograms. The mass of the planet is on the order of millions of millions of millions of millions of kilograms.



The other part of the question is about why removing this mass doesn't affect Earth's orbit. A, possibly surprising, fact is that even if you decreased the mass of the planet by a significant amount you would not actually change its orbit. That is because mass comes in numerators and denominators in the relevant equations, and cancels out.



You mention "then gravitational attraction between earth and sun must decrease". That is true. If you halved the mass of the Earth, you would get half the gravitational force on it. However, because force is mass times acceleration, when you halve force and you halve mass, you are left with the same acceleration. The other fact is that the momentum of the Earth decreases when this mass decreases. Again that is true: momentum is proportional to mass times velocity. Once again though, that does not mean that it's going any slower. There is just less of it going as fast as before.



Fundamentally, orbital radius is a function of tangential velocity and centripedal acceleration, and neither of those has changed. Therefore even if we had launched an impossibly huge proportion of the planet (assuming we put it in orbit or disperse it equally in all directions) that wouldn't cause the planet to orbit further from the sun.







share|cite|improve this answer








New contributor



Josiah is a new contributor to this site. Take care in asking for clarification, commenting, and answering.
Check out our Code of Conduct.








share|cite|improve this answer



share|cite|improve this answer






New contributor



Josiah is a new contributor to this site. Take care in asking for clarification, commenting, and answering.
Check out our Code of Conduct.








answered Jun 16 at 9:03









JosiahJosiah

1513 bronze badges




1513 bronze badges




New contributor



Josiah is a new contributor to this site. Take care in asking for clarification, commenting, and answering.
Check out our Code of Conduct.




New contributor




Josiah is a new contributor to this site. Take care in asking for clarification, commenting, and answering.
Check out our Code of Conduct.









  • 2




    $begingroup$
    "mass comes in numerators and denominators in the relevant equations, and cancels out." Almost. See en.wikipedia.org/wiki/… But practically speaking, Earth's mass is very small compared to the Sun, so tiny variations of Earth's mass can safely be ignored when calculating its orbit.
    $endgroup$
    – PM 2Ring
    Jun 16 at 9:27












  • 2




    $begingroup$
    "mass comes in numerators and denominators in the relevant equations, and cancels out." Almost. See en.wikipedia.org/wiki/… But practically speaking, Earth's mass is very small compared to the Sun, so tiny variations of Earth's mass can safely be ignored when calculating its orbit.
    $endgroup$
    – PM 2Ring
    Jun 16 at 9:27







2




2




$begingroup$
"mass comes in numerators and denominators in the relevant equations, and cancels out." Almost. See en.wikipedia.org/wiki/… But practically speaking, Earth's mass is very small compared to the Sun, so tiny variations of Earth's mass can safely be ignored when calculating its orbit.
$endgroup$
– PM 2Ring
Jun 16 at 9:27




$begingroup$
"mass comes in numerators and denominators in the relevant equations, and cancels out." Almost. See en.wikipedia.org/wiki/… But practically speaking, Earth's mass is very small compared to the Sun, so tiny variations of Earth's mass can safely be ignored when calculating its orbit.
$endgroup$
– PM 2Ring
Jun 16 at 9:27











5












$begingroup$

All the existing answers nicely explain the issue, but I'll add some numbers to the discussion:



In 2018, 112 objects have been launched to orbit. Let's assume a typical rocket mass of 500 t and a payload of 10 t (values for the Falcon 9). Of these 112 objects, 30 were launched into high orbits where part of the rocket exhaust may have escaped Earth's sphere of influence (SOI), and four (Elon Musk's Tesla Roadster, InSight, Parker Solar Probe, and BepiColombo) were launched into interplanetary orbits where they and their upper rocket stages, escaped Earth's SOI themselves.



For high orbits, we can assume that about half of the payload's mass leaves Earth's SOI in the form of rocket exhaust. For payloads that are launched into interplanetary space, we can assume that an additional 10 tons are sent in the form of the launcher's spent upper stage.



Assuming that 2018 is a representative year and those guesses are about correct (variations from 0.1 to 10 would be possible),



  • The total launched mass is $5 cdot 10^7 kg/year$

  • The total satellite mass increases by $1 cdot 10^6 kg/year$

  • The total satellite mass in high orbits increases by $3 cdot 10^5 kg/year$

  • The total mass leaving Earth's SOI, from both interplanetary launches and rocket exhaust of high-orbit satellites, is $3 cdot 10^5 kg/year$

As discussed in the other answers, the mass of satellites that stay inside Earth's SOI has no influence on the distance between Earth and the Sun. The only relevant part is the $3 cdot 10^5 kg/year$ that are launched away from Earth.



As you correctly noted, the mass itself isn't relevant, the impulse is, so let's assume that the mass leaves Earth at 10 km/s. The fastest-ever spacecraft to leave Earth was New Horizons at 16 km/s, while a typical Mars transfer takes 6 km/s. Rocket exhaust is typically much slower, on the order of 3 km/s.



We arrive at an impulse of $ 3 cdot 10^9 Ns / year $, or an average force of $ 100 N $.



But in which direction is the force applied? For interplanetary probes the ejection direction depends on your intended destination and transfer plan, while for rocket exhaust from satellites in high orbits it's pretty much random. All in all, I guess that it will pretty much even out. After all, in 2018 two launches were to higher solar orbits and two were to lower solar orbits.



If the force is applied in the same direction in which Earth is moving, it is accelerated and thus lifted into a higher orbit. If the force is applied in the opposite direction, it is decelerated and moves into a lower orbit (closer to the Sun).



The orbital energy of Earth is calculated as $
-frac
G cdot M cdot m_Earth2a$
, where G is the constant of gravity, M is the total mass of the Earth-Sun system, and a is the semi-major axis (average distance between Earth and Sun).



The power at which Earth is accelerated or decelerated is $fracdEdt = Fv$, where F is the force of acceleration and v is the orbital velocity of Earth. We arrive at



$a(t) = - fracG cdot M cdot m_Earth2 cdot (E_0 pm F v t) = - fracG cdot M cdot m_Earth2 cdot E_0 cdot frac11 pm cfracF vE_0 cdot t $



$fracda(t)dt = - fracG cdot M cdot m_Earth2 cdot E_0 cdot fracda(t)dt (frac11 pm fracF vE_0 cdot t) = - fracG cdot M cdot m_Earth2 cdot E_0 cdot fracmp fracF vE_0(1 pm fracF vE_0 cdot t)^2 $.



$fracF vE_0 cdot t$ is negligible compared to 1 (about $t cdot 3 cdot 10^-20 / year$), so the formula is simplified to:



$fracdadt = pm fracG cdot M cdot m_Earth2 cdot E_0 cdot fracF vE_0 = pm GM cdot fracm_Earthcdot v2 cdot E_0^2 cdot F = 1.8 cdot 10^-18 fracfracmsN cdot F$.



So the result on Earth's distance from the Sun is $pm 1.8 cdot 10^-16 m/s = pm 5 cdot 10^-9 m/year$, assuming that everything is ejected in the same direction.






share|cite|improve this answer











$endgroup$








  • 1




    $begingroup$
    Finally some numbers instead of all the hand waving!
    $endgroup$
    – Peter Mortensen
    Jun 17 at 7:46















5












$begingroup$

All the existing answers nicely explain the issue, but I'll add some numbers to the discussion:



In 2018, 112 objects have been launched to orbit. Let's assume a typical rocket mass of 500 t and a payload of 10 t (values for the Falcon 9). Of these 112 objects, 30 were launched into high orbits where part of the rocket exhaust may have escaped Earth's sphere of influence (SOI), and four (Elon Musk's Tesla Roadster, InSight, Parker Solar Probe, and BepiColombo) were launched into interplanetary orbits where they and their upper rocket stages, escaped Earth's SOI themselves.



For high orbits, we can assume that about half of the payload's mass leaves Earth's SOI in the form of rocket exhaust. For payloads that are launched into interplanetary space, we can assume that an additional 10 tons are sent in the form of the launcher's spent upper stage.



Assuming that 2018 is a representative year and those guesses are about correct (variations from 0.1 to 10 would be possible),



  • The total launched mass is $5 cdot 10^7 kg/year$

  • The total satellite mass increases by $1 cdot 10^6 kg/year$

  • The total satellite mass in high orbits increases by $3 cdot 10^5 kg/year$

  • The total mass leaving Earth's SOI, from both interplanetary launches and rocket exhaust of high-orbit satellites, is $3 cdot 10^5 kg/year$

As discussed in the other answers, the mass of satellites that stay inside Earth's SOI has no influence on the distance between Earth and the Sun. The only relevant part is the $3 cdot 10^5 kg/year$ that are launched away from Earth.



As you correctly noted, the mass itself isn't relevant, the impulse is, so let's assume that the mass leaves Earth at 10 km/s. The fastest-ever spacecraft to leave Earth was New Horizons at 16 km/s, while a typical Mars transfer takes 6 km/s. Rocket exhaust is typically much slower, on the order of 3 km/s.



We arrive at an impulse of $ 3 cdot 10^9 Ns / year $, or an average force of $ 100 N $.



But in which direction is the force applied? For interplanetary probes the ejection direction depends on your intended destination and transfer plan, while for rocket exhaust from satellites in high orbits it's pretty much random. All in all, I guess that it will pretty much even out. After all, in 2018 two launches were to higher solar orbits and two were to lower solar orbits.



If the force is applied in the same direction in which Earth is moving, it is accelerated and thus lifted into a higher orbit. If the force is applied in the opposite direction, it is decelerated and moves into a lower orbit (closer to the Sun).



The orbital energy of Earth is calculated as $
-frac
G cdot M cdot m_Earth2a$
, where G is the constant of gravity, M is the total mass of the Earth-Sun system, and a is the semi-major axis (average distance between Earth and Sun).



The power at which Earth is accelerated or decelerated is $fracdEdt = Fv$, where F is the force of acceleration and v is the orbital velocity of Earth. We arrive at



$a(t) = - fracG cdot M cdot m_Earth2 cdot (E_0 pm F v t) = - fracG cdot M cdot m_Earth2 cdot E_0 cdot frac11 pm cfracF vE_0 cdot t $



$fracda(t)dt = - fracG cdot M cdot m_Earth2 cdot E_0 cdot fracda(t)dt (frac11 pm fracF vE_0 cdot t) = - fracG cdot M cdot m_Earth2 cdot E_0 cdot fracmp fracF vE_0(1 pm fracF vE_0 cdot t)^2 $.



$fracF vE_0 cdot t$ is negligible compared to 1 (about $t cdot 3 cdot 10^-20 / year$), so the formula is simplified to:



$fracdadt = pm fracG cdot M cdot m_Earth2 cdot E_0 cdot fracF vE_0 = pm GM cdot fracm_Earthcdot v2 cdot E_0^2 cdot F = 1.8 cdot 10^-18 fracfracmsN cdot F$.



So the result on Earth's distance from the Sun is $pm 1.8 cdot 10^-16 m/s = pm 5 cdot 10^-9 m/year$, assuming that everything is ejected in the same direction.






share|cite|improve this answer











$endgroup$








  • 1




    $begingroup$
    Finally some numbers instead of all the hand waving!
    $endgroup$
    – Peter Mortensen
    Jun 17 at 7:46













5












5








5





$begingroup$

All the existing answers nicely explain the issue, but I'll add some numbers to the discussion:



In 2018, 112 objects have been launched to orbit. Let's assume a typical rocket mass of 500 t and a payload of 10 t (values for the Falcon 9). Of these 112 objects, 30 were launched into high orbits where part of the rocket exhaust may have escaped Earth's sphere of influence (SOI), and four (Elon Musk's Tesla Roadster, InSight, Parker Solar Probe, and BepiColombo) were launched into interplanetary orbits where they and their upper rocket stages, escaped Earth's SOI themselves.



For high orbits, we can assume that about half of the payload's mass leaves Earth's SOI in the form of rocket exhaust. For payloads that are launched into interplanetary space, we can assume that an additional 10 tons are sent in the form of the launcher's spent upper stage.



Assuming that 2018 is a representative year and those guesses are about correct (variations from 0.1 to 10 would be possible),



  • The total launched mass is $5 cdot 10^7 kg/year$

  • The total satellite mass increases by $1 cdot 10^6 kg/year$

  • The total satellite mass in high orbits increases by $3 cdot 10^5 kg/year$

  • The total mass leaving Earth's SOI, from both interplanetary launches and rocket exhaust of high-orbit satellites, is $3 cdot 10^5 kg/year$

As discussed in the other answers, the mass of satellites that stay inside Earth's SOI has no influence on the distance between Earth and the Sun. The only relevant part is the $3 cdot 10^5 kg/year$ that are launched away from Earth.



As you correctly noted, the mass itself isn't relevant, the impulse is, so let's assume that the mass leaves Earth at 10 km/s. The fastest-ever spacecraft to leave Earth was New Horizons at 16 km/s, while a typical Mars transfer takes 6 km/s. Rocket exhaust is typically much slower, on the order of 3 km/s.



We arrive at an impulse of $ 3 cdot 10^9 Ns / year $, or an average force of $ 100 N $.



But in which direction is the force applied? For interplanetary probes the ejection direction depends on your intended destination and transfer plan, while for rocket exhaust from satellites in high orbits it's pretty much random. All in all, I guess that it will pretty much even out. After all, in 2018 two launches were to higher solar orbits and two were to lower solar orbits.



If the force is applied in the same direction in which Earth is moving, it is accelerated and thus lifted into a higher orbit. If the force is applied in the opposite direction, it is decelerated and moves into a lower orbit (closer to the Sun).



The orbital energy of Earth is calculated as $
-frac
G cdot M cdot m_Earth2a$
, where G is the constant of gravity, M is the total mass of the Earth-Sun system, and a is the semi-major axis (average distance between Earth and Sun).



The power at which Earth is accelerated or decelerated is $fracdEdt = Fv$, where F is the force of acceleration and v is the orbital velocity of Earth. We arrive at



$a(t) = - fracG cdot M cdot m_Earth2 cdot (E_0 pm F v t) = - fracG cdot M cdot m_Earth2 cdot E_0 cdot frac11 pm cfracF vE_0 cdot t $



$fracda(t)dt = - fracG cdot M cdot m_Earth2 cdot E_0 cdot fracda(t)dt (frac11 pm fracF vE_0 cdot t) = - fracG cdot M cdot m_Earth2 cdot E_0 cdot fracmp fracF vE_0(1 pm fracF vE_0 cdot t)^2 $.



$fracF vE_0 cdot t$ is negligible compared to 1 (about $t cdot 3 cdot 10^-20 / year$), so the formula is simplified to:



$fracdadt = pm fracG cdot M cdot m_Earth2 cdot E_0 cdot fracF vE_0 = pm GM cdot fracm_Earthcdot v2 cdot E_0^2 cdot F = 1.8 cdot 10^-18 fracfracmsN cdot F$.



So the result on Earth's distance from the Sun is $pm 1.8 cdot 10^-16 m/s = pm 5 cdot 10^-9 m/year$, assuming that everything is ejected in the same direction.






share|cite|improve this answer











$endgroup$



All the existing answers nicely explain the issue, but I'll add some numbers to the discussion:



In 2018, 112 objects have been launched to orbit. Let's assume a typical rocket mass of 500 t and a payload of 10 t (values for the Falcon 9). Of these 112 objects, 30 were launched into high orbits where part of the rocket exhaust may have escaped Earth's sphere of influence (SOI), and four (Elon Musk's Tesla Roadster, InSight, Parker Solar Probe, and BepiColombo) were launched into interplanetary orbits where they and their upper rocket stages, escaped Earth's SOI themselves.



For high orbits, we can assume that about half of the payload's mass leaves Earth's SOI in the form of rocket exhaust. For payloads that are launched into interplanetary space, we can assume that an additional 10 tons are sent in the form of the launcher's spent upper stage.



Assuming that 2018 is a representative year and those guesses are about correct (variations from 0.1 to 10 would be possible),



  • The total launched mass is $5 cdot 10^7 kg/year$

  • The total satellite mass increases by $1 cdot 10^6 kg/year$

  • The total satellite mass in high orbits increases by $3 cdot 10^5 kg/year$

  • The total mass leaving Earth's SOI, from both interplanetary launches and rocket exhaust of high-orbit satellites, is $3 cdot 10^5 kg/year$

As discussed in the other answers, the mass of satellites that stay inside Earth's SOI has no influence on the distance between Earth and the Sun. The only relevant part is the $3 cdot 10^5 kg/year$ that are launched away from Earth.



As you correctly noted, the mass itself isn't relevant, the impulse is, so let's assume that the mass leaves Earth at 10 km/s. The fastest-ever spacecraft to leave Earth was New Horizons at 16 km/s, while a typical Mars transfer takes 6 km/s. Rocket exhaust is typically much slower, on the order of 3 km/s.



We arrive at an impulse of $ 3 cdot 10^9 Ns / year $, or an average force of $ 100 N $.



But in which direction is the force applied? For interplanetary probes the ejection direction depends on your intended destination and transfer plan, while for rocket exhaust from satellites in high orbits it's pretty much random. All in all, I guess that it will pretty much even out. After all, in 2018 two launches were to higher solar orbits and two were to lower solar orbits.



If the force is applied in the same direction in which Earth is moving, it is accelerated and thus lifted into a higher orbit. If the force is applied in the opposite direction, it is decelerated and moves into a lower orbit (closer to the Sun).



The orbital energy of Earth is calculated as $
-frac
G cdot M cdot m_Earth2a$
, where G is the constant of gravity, M is the total mass of the Earth-Sun system, and a is the semi-major axis (average distance between Earth and Sun).



The power at which Earth is accelerated or decelerated is $fracdEdt = Fv$, where F is the force of acceleration and v is the orbital velocity of Earth. We arrive at



$a(t) = - fracG cdot M cdot m_Earth2 cdot (E_0 pm F v t) = - fracG cdot M cdot m_Earth2 cdot E_0 cdot frac11 pm cfracF vE_0 cdot t $



$fracda(t)dt = - fracG cdot M cdot m_Earth2 cdot E_0 cdot fracda(t)dt (frac11 pm fracF vE_0 cdot t) = - fracG cdot M cdot m_Earth2 cdot E_0 cdot fracmp fracF vE_0(1 pm fracF vE_0 cdot t)^2 $.



$fracF vE_0 cdot t$ is negligible compared to 1 (about $t cdot 3 cdot 10^-20 / year$), so the formula is simplified to:



$fracdadt = pm fracG cdot M cdot m_Earth2 cdot E_0 cdot fracF vE_0 = pm GM cdot fracm_Earthcdot v2 cdot E_0^2 cdot F = 1.8 cdot 10^-18 fracfracmsN cdot F$.



So the result on Earth's distance from the Sun is $pm 1.8 cdot 10^-16 m/s = pm 5 cdot 10^-9 m/year$, assuming that everything is ejected in the same direction.







share|cite|improve this answer














share|cite|improve this answer



share|cite|improve this answer








edited Jun 17 at 16:46

























answered Jun 16 at 15:25









mic_emic_e

1614 bronze badges




1614 bronze badges







  • 1




    $begingroup$
    Finally some numbers instead of all the hand waving!
    $endgroup$
    – Peter Mortensen
    Jun 17 at 7:46












  • 1




    $begingroup$
    Finally some numbers instead of all the hand waving!
    $endgroup$
    – Peter Mortensen
    Jun 17 at 7:46







1




1




$begingroup$
Finally some numbers instead of all the hand waving!
$endgroup$
– Peter Mortensen
Jun 17 at 7:46




$begingroup$
Finally some numbers instead of all the hand waving!
$endgroup$
– Peter Mortensen
Jun 17 at 7:46



Popular posts from this blog

Get product attribute by attribute group code in magento 2get product attribute by product attribute group in magento 2Magento 2 Log Bundle Product Data in List Page?How to get all product attribute of a attribute group of Default attribute set?Magento 2.1 Create a filter in the product grid by new attributeMagento 2 : Get Product Attribute values By GroupMagento 2 How to get all existing values for one attributeMagento 2 get custom attribute of a single product inside a pluginMagento 2.3 How to get all the Multi Source Inventory (MSI) locations collection in custom module?Magento2: how to develop rest API to get new productsGet product attribute by attribute group code ( [attribute_group_code] ) in magento 2

Category:9 (number) SubcategoriesMedia in category "9 (number)"Navigation menuUpload mediaGND ID: 4485639-8Library of Congress authority ID: sh85091979ReasonatorScholiaStatistics

Magento 2.3: How do i solve this, Not registered handle, on custom form?How can i rewrite TierPrice Block in Magento2magento 2 captcha not rendering if I override layout xmlmain.CRITICAL: Plugin class doesn't existMagento 2 : Problem while adding custom button order view page?Magento 2.2.5: Overriding Admin Controller sales/orderMagento 2.2.5: Add, Update and Delete existing products Custom OptionsMagento 2.3 : File Upload issue in UI Component FormMagento2 Not registered handleHow to configured Form Builder Js in my custom magento 2.3.0 module?Magento 2.3. How to create image upload field in an admin form